Is this bounded convex set in $mathbb{R}^n$ closed?











up vote
2
down vote

favorite












Suppose we have a bounded set $C subset mathbb{R}^n$ that is convex and non-empty.
And suppose the family of linear functions $(f_{x})_{x in mathbb{R}^n}$ given by $f_{x}: C rightarrow mathbb{R}$, $ f_{x}(c) = x.c$ for $c in C$ attain their maximum and minimum in the set $C$.



Does this mean $C$ is closed (and hence compact) in $mathbb{R}^n$?



My idea: I think this does imply $C$ is closed but I am not sure how to write my argument "properly". For every vector $x in mathbb{R}^n$, there is a point in $C$ that is "furthest" in the direction of $x$. Then because we are in a convex set we can just "join up all these points" and our set is closed. But how do I write this formally.



Remark: Also is it true that if a linear function on a convex set attains its maximum/minimum it does so on the boundary?










share|cite|improve this question
























  • Note that the argument used in this very interesting question math.stackexchange.com/questions/1434741/… may be of some help.....
    – Namch96
    Nov 14 at 16:25










  • Is $x.c$ the inner product between $x$ and $c$? The answer to the last question is yes due to the maximum principle.
    – LinAlg
    Nov 14 at 18:38












  • Yes, it is the inner product.
    – Namch96
    Nov 14 at 23:49















up vote
2
down vote

favorite












Suppose we have a bounded set $C subset mathbb{R}^n$ that is convex and non-empty.
And suppose the family of linear functions $(f_{x})_{x in mathbb{R}^n}$ given by $f_{x}: C rightarrow mathbb{R}$, $ f_{x}(c) = x.c$ for $c in C$ attain their maximum and minimum in the set $C$.



Does this mean $C$ is closed (and hence compact) in $mathbb{R}^n$?



My idea: I think this does imply $C$ is closed but I am not sure how to write my argument "properly". For every vector $x in mathbb{R}^n$, there is a point in $C$ that is "furthest" in the direction of $x$. Then because we are in a convex set we can just "join up all these points" and our set is closed. But how do I write this formally.



Remark: Also is it true that if a linear function on a convex set attains its maximum/minimum it does so on the boundary?










share|cite|improve this question
























  • Note that the argument used in this very interesting question math.stackexchange.com/questions/1434741/… may be of some help.....
    – Namch96
    Nov 14 at 16:25










  • Is $x.c$ the inner product between $x$ and $c$? The answer to the last question is yes due to the maximum principle.
    – LinAlg
    Nov 14 at 18:38












  • Yes, it is the inner product.
    – Namch96
    Nov 14 at 23:49













up vote
2
down vote

favorite









up vote
2
down vote

favorite











Suppose we have a bounded set $C subset mathbb{R}^n$ that is convex and non-empty.
And suppose the family of linear functions $(f_{x})_{x in mathbb{R}^n}$ given by $f_{x}: C rightarrow mathbb{R}$, $ f_{x}(c) = x.c$ for $c in C$ attain their maximum and minimum in the set $C$.



Does this mean $C$ is closed (and hence compact) in $mathbb{R}^n$?



My idea: I think this does imply $C$ is closed but I am not sure how to write my argument "properly". For every vector $x in mathbb{R}^n$, there is a point in $C$ that is "furthest" in the direction of $x$. Then because we are in a convex set we can just "join up all these points" and our set is closed. But how do I write this formally.



Remark: Also is it true that if a linear function on a convex set attains its maximum/minimum it does so on the boundary?










share|cite|improve this question















Suppose we have a bounded set $C subset mathbb{R}^n$ that is convex and non-empty.
And suppose the family of linear functions $(f_{x})_{x in mathbb{R}^n}$ given by $f_{x}: C rightarrow mathbb{R}$, $ f_{x}(c) = x.c$ for $c in C$ attain their maximum and minimum in the set $C$.



Does this mean $C$ is closed (and hence compact) in $mathbb{R}^n$?



My idea: I think this does imply $C$ is closed but I am not sure how to write my argument "properly". For every vector $x in mathbb{R}^n$, there is a point in $C$ that is "furthest" in the direction of $x$. Then because we are in a convex set we can just "join up all these points" and our set is closed. But how do I write this formally.



Remark: Also is it true that if a linear function on a convex set attains its maximum/minimum it does so on the boundary?







convex-analysis convex-optimization compactness






share|cite|improve this question















share|cite|improve this question













share|cite|improve this question




share|cite|improve this question








edited Nov 14 at 21:46

























asked Nov 14 at 16:25









Namch96

449314




449314












  • Note that the argument used in this very interesting question math.stackexchange.com/questions/1434741/… may be of some help.....
    – Namch96
    Nov 14 at 16:25










  • Is $x.c$ the inner product between $x$ and $c$? The answer to the last question is yes due to the maximum principle.
    – LinAlg
    Nov 14 at 18:38












  • Yes, it is the inner product.
    – Namch96
    Nov 14 at 23:49


















  • Note that the argument used in this very interesting question math.stackexchange.com/questions/1434741/… may be of some help.....
    – Namch96
    Nov 14 at 16:25










  • Is $x.c$ the inner product between $x$ and $c$? The answer to the last question is yes due to the maximum principle.
    – LinAlg
    Nov 14 at 18:38












  • Yes, it is the inner product.
    – Namch96
    Nov 14 at 23:49
















Note that the argument used in this very interesting question math.stackexchange.com/questions/1434741/… may be of some help.....
– Namch96
Nov 14 at 16:25




Note that the argument used in this very interesting question math.stackexchange.com/questions/1434741/… may be of some help.....
– Namch96
Nov 14 at 16:25












Is $x.c$ the inner product between $x$ and $c$? The answer to the last question is yes due to the maximum principle.
– LinAlg
Nov 14 at 18:38






Is $x.c$ the inner product between $x$ and $c$? The answer to the last question is yes due to the maximum principle.
– LinAlg
Nov 14 at 18:38














Yes, it is the inner product.
– Namch96
Nov 14 at 23:49




Yes, it is the inner product.
– Namch96
Nov 14 at 23:49










1 Answer
1






active

oldest

votes

















up vote
2
down vote



accepted










I found a counterexample as I was trying to construct a proof. Let me explain my thought process.



Let $x'$ be on the boundary and assume $x'notin C$. Since $x'$ is on the boundary, it has a separating hyperplane. Let $c$ be perpendicular to that plane and consider the function $f(x)=c^Tx$. Since $x'$ is on the boundary, function values can get arbitrarily close to $c^Tx'$, but by construction, cannot exceed $c^Tx'$ (the separating hyperplane is an isocurve). Since $f$ attains its maximum on $C$, there is a point $x^* in C$ for which $c^Tx' = c^Tx^*$. So, $x^*$ and $x'$ share the same separating hyperplane.



The next step in my proof would be to construct a point on the other side of $x'$ that is in $C$ and reach a contradiction with convexity. This is where I had the aha moment.



Consider the following set, which is a closed rectangle and the first quadrant of a closed circle, minus the point where the rectangle meets the circle.



enter image description here






share|cite|improve this answer





















  • Very nice, thanks!
    – Namch96
    Nov 15 at 14:15










  • Yes, nice. I had gotten to the point of thinking about separating hyperplanes and started to doubt the claim but couldn't come up with a counterexample. Yours is excellent.
    – JonathanZ
    Nov 17 at 16:10













Your Answer





StackExchange.ifUsing("editor", function () {
return StackExchange.using("mathjaxEditing", function () {
StackExchange.MarkdownEditor.creationCallbacks.add(function (editor, postfix) {
StackExchange.mathjaxEditing.prepareWmdForMathJax(editor, postfix, [["$", "$"], ["\\(","\\)"]]);
});
});
}, "mathjax-editing");

StackExchange.ready(function() {
var channelOptions = {
tags: "".split(" "),
id: "69"
};
initTagRenderer("".split(" "), "".split(" "), channelOptions);

StackExchange.using("externalEditor", function() {
// Have to fire editor after snippets, if snippets enabled
if (StackExchange.settings.snippets.snippetsEnabled) {
StackExchange.using("snippets", function() {
createEditor();
});
}
else {
createEditor();
}
});

function createEditor() {
StackExchange.prepareEditor({
heartbeatType: 'answer',
convertImagesToLinks: true,
noModals: true,
showLowRepImageUploadWarning: true,
reputationToPostImages: 10,
bindNavPrevention: true,
postfix: "",
imageUploader: {
brandingHtml: "Powered by u003ca class="icon-imgur-white" href="https://imgur.com/"u003eu003c/au003e",
contentPolicyHtml: "User contributions licensed under u003ca href="https://creativecommons.org/licenses/by-sa/3.0/"u003ecc by-sa 3.0 with attribution requiredu003c/au003e u003ca href="https://stackoverflow.com/legal/content-policy"u003e(content policy)u003c/au003e",
allowUrls: true
},
noCode: true, onDemand: true,
discardSelector: ".discard-answer"
,immediatelyShowMarkdownHelp:true
});


}
});














 

draft saved


draft discarded


















StackExchange.ready(
function () {
StackExchange.openid.initPostLogin('.new-post-login', 'https%3a%2f%2fmath.stackexchange.com%2fquestions%2f2998469%2fis-this-bounded-convex-set-in-mathbbrn-closed%23new-answer', 'question_page');
}
);

Post as a guest















Required, but never shown

























1 Answer
1






active

oldest

votes








1 Answer
1






active

oldest

votes









active

oldest

votes






active

oldest

votes








up vote
2
down vote



accepted










I found a counterexample as I was trying to construct a proof. Let me explain my thought process.



Let $x'$ be on the boundary and assume $x'notin C$. Since $x'$ is on the boundary, it has a separating hyperplane. Let $c$ be perpendicular to that plane and consider the function $f(x)=c^Tx$. Since $x'$ is on the boundary, function values can get arbitrarily close to $c^Tx'$, but by construction, cannot exceed $c^Tx'$ (the separating hyperplane is an isocurve). Since $f$ attains its maximum on $C$, there is a point $x^* in C$ for which $c^Tx' = c^Tx^*$. So, $x^*$ and $x'$ share the same separating hyperplane.



The next step in my proof would be to construct a point on the other side of $x'$ that is in $C$ and reach a contradiction with convexity. This is where I had the aha moment.



Consider the following set, which is a closed rectangle and the first quadrant of a closed circle, minus the point where the rectangle meets the circle.



enter image description here






share|cite|improve this answer





















  • Very nice, thanks!
    – Namch96
    Nov 15 at 14:15










  • Yes, nice. I had gotten to the point of thinking about separating hyperplanes and started to doubt the claim but couldn't come up with a counterexample. Yours is excellent.
    – JonathanZ
    Nov 17 at 16:10

















up vote
2
down vote



accepted










I found a counterexample as I was trying to construct a proof. Let me explain my thought process.



Let $x'$ be on the boundary and assume $x'notin C$. Since $x'$ is on the boundary, it has a separating hyperplane. Let $c$ be perpendicular to that plane and consider the function $f(x)=c^Tx$. Since $x'$ is on the boundary, function values can get arbitrarily close to $c^Tx'$, but by construction, cannot exceed $c^Tx'$ (the separating hyperplane is an isocurve). Since $f$ attains its maximum on $C$, there is a point $x^* in C$ for which $c^Tx' = c^Tx^*$. So, $x^*$ and $x'$ share the same separating hyperplane.



The next step in my proof would be to construct a point on the other side of $x'$ that is in $C$ and reach a contradiction with convexity. This is where I had the aha moment.



Consider the following set, which is a closed rectangle and the first quadrant of a closed circle, minus the point where the rectangle meets the circle.



enter image description here






share|cite|improve this answer





















  • Very nice, thanks!
    – Namch96
    Nov 15 at 14:15










  • Yes, nice. I had gotten to the point of thinking about separating hyperplanes and started to doubt the claim but couldn't come up with a counterexample. Yours is excellent.
    – JonathanZ
    Nov 17 at 16:10















up vote
2
down vote



accepted







up vote
2
down vote



accepted






I found a counterexample as I was trying to construct a proof. Let me explain my thought process.



Let $x'$ be on the boundary and assume $x'notin C$. Since $x'$ is on the boundary, it has a separating hyperplane. Let $c$ be perpendicular to that plane and consider the function $f(x)=c^Tx$. Since $x'$ is on the boundary, function values can get arbitrarily close to $c^Tx'$, but by construction, cannot exceed $c^Tx'$ (the separating hyperplane is an isocurve). Since $f$ attains its maximum on $C$, there is a point $x^* in C$ for which $c^Tx' = c^Tx^*$. So, $x^*$ and $x'$ share the same separating hyperplane.



The next step in my proof would be to construct a point on the other side of $x'$ that is in $C$ and reach a contradiction with convexity. This is where I had the aha moment.



Consider the following set, which is a closed rectangle and the first quadrant of a closed circle, minus the point where the rectangle meets the circle.



enter image description here






share|cite|improve this answer












I found a counterexample as I was trying to construct a proof. Let me explain my thought process.



Let $x'$ be on the boundary and assume $x'notin C$. Since $x'$ is on the boundary, it has a separating hyperplane. Let $c$ be perpendicular to that plane and consider the function $f(x)=c^Tx$. Since $x'$ is on the boundary, function values can get arbitrarily close to $c^Tx'$, but by construction, cannot exceed $c^Tx'$ (the separating hyperplane is an isocurve). Since $f$ attains its maximum on $C$, there is a point $x^* in C$ for which $c^Tx' = c^Tx^*$. So, $x^*$ and $x'$ share the same separating hyperplane.



The next step in my proof would be to construct a point on the other side of $x'$ that is in $C$ and reach a contradiction with convexity. This is where I had the aha moment.



Consider the following set, which is a closed rectangle and the first quadrant of a closed circle, minus the point where the rectangle meets the circle.



enter image description here







share|cite|improve this answer












share|cite|improve this answer



share|cite|improve this answer










answered Nov 15 at 2:02









LinAlg

7,6241520




7,6241520












  • Very nice, thanks!
    – Namch96
    Nov 15 at 14:15










  • Yes, nice. I had gotten to the point of thinking about separating hyperplanes and started to doubt the claim but couldn't come up with a counterexample. Yours is excellent.
    – JonathanZ
    Nov 17 at 16:10




















  • Very nice, thanks!
    – Namch96
    Nov 15 at 14:15










  • Yes, nice. I had gotten to the point of thinking about separating hyperplanes and started to doubt the claim but couldn't come up with a counterexample. Yours is excellent.
    – JonathanZ
    Nov 17 at 16:10


















Very nice, thanks!
– Namch96
Nov 15 at 14:15




Very nice, thanks!
– Namch96
Nov 15 at 14:15












Yes, nice. I had gotten to the point of thinking about separating hyperplanes and started to doubt the claim but couldn't come up with a counterexample. Yours is excellent.
– JonathanZ
Nov 17 at 16:10






Yes, nice. I had gotten to the point of thinking about separating hyperplanes and started to doubt the claim but couldn't come up with a counterexample. Yours is excellent.
– JonathanZ
Nov 17 at 16:10




















 

draft saved


draft discarded



















































 


draft saved


draft discarded














StackExchange.ready(
function () {
StackExchange.openid.initPostLogin('.new-post-login', 'https%3a%2f%2fmath.stackexchange.com%2fquestions%2f2998469%2fis-this-bounded-convex-set-in-mathbbrn-closed%23new-answer', 'question_page');
}
);

Post as a guest















Required, but never shown





















































Required, but never shown














Required, but never shown












Required, but never shown







Required, but never shown

































Required, but never shown














Required, but never shown












Required, but never shown







Required, but never shown







Popular posts from this blog

Plaza Victoria

In PowerPoint, is there a keyboard shortcut for bulleted / numbered list?

How to put 3 figures in Latex with 2 figures side by side and 1 below these side by side images but in...